MathGroup Archive 2007

[Date Index] [Thread Index] [Author Index]

Search the Archive

A limit bug

  • To: mathgroup at smc.vnet.net
  • Subject: [mg84511] A limit bug
  • From: "David W.Cantrell" <DWCantrell at sigmaxi.net>
  • Date: Mon, 31 Dec 2007 21:16:27 -0500 (EST)

A recent question in sci.math led to something which should also interest
this group.

The OP asked about the limit of (p + q)!/(p! q!) as both p and q increase
without bound. And he said later

> I wasn't sure about it because Mathematica gives me a limit of zero.
> Isn't that strange?

I responded as follows.

----------------------------------------------

Well, it's a bug. I suppose that what you did in Mathematica was something
like

In[3]:= Limit[Limit[(p + q)!/(p! q!), q -> Infinity], p -> Infinity]

Out[3]= 0

But note that there is not any way -- well, at least none known to me -- in
Mathematica to get a true general "two-variable" limit:

limit f(x,y) as (x,y) -> (x0,y0)

However, Mathematica can get a correct answer for your limit problem.

First, realize that (p + q)!/(p! q!) is Multinomial[p, q]. So you might try

In[5]:= Limit[Limit[Multinomial[p, q], q -> Infinity], p -> Infinity]

Out[5]= Limit[Limit[Multinomial[p, q], q -> Infinity], p -> Infinity]

Since that remains unevaluated (but at least there was now no bug!), you
might consider the possibility that it remained unevaluated for a good
reason, namely, because a little more information had to be provided:

In[6]:= Limit[Limit[Multinomial[p, q], q -> Infinity, Assumptions -> p > 1],
p -> Infinity]

Out[6]= Infinity

Success! Happy New Year!

But BTW, note that, curiously, the following fails:

In[7]:= Limit[Limit[(p + q)!/(p! q!), q -> Infinity, Assumptions -> p > 1],
p -> Infinity]

Out[7]= Indeterminate

David


  • Prev by Date: Re: Unevaluated limit
  • Next by Date: Integrating "If"
  • Previous by thread: small init.m problem
  • Next by thread: Integrating "If"